LSAT and Law School Admissions Forum

Get expert LSAT preparation and law school admissions advice from PowerScore Test Preparation.

 Administrator
PowerScore Staff
  • PowerScore Staff
  • Posts: 8916
  • Joined: Feb 02, 2011
|
#41264
Complete Question Explanation
(The complete setup for this game can be found here: lsat/viewtopic.php?t=6641)

The correct answer choice is (A)

To minimize the number of times W plays a T, focus on Template #2, which has no immediate assignment of W to a T (Template #1 already has W playing a T fourth). Next, attempt to minimize the number of times W performs a T in Template #1.

First, can both the second and third solos (both T) be played by Z? Yes:
PT66_J12_Game_#4_#20_diagram 1.png
Next, from the fourth rule, because Z plays the second solo, W must play the fifth solo. Can the fifth solo then not be a T? Yes, it can be an M:
PT66_J12_Game_#4_#20_diagram 2.png
The hypothetical above conforms to all of the rules, and because W does not play a T at any point, the correct answer is zero. Thus, answer choice (A) is correct.
You do not have the required permissions to view the files attached to this post.
 karlaurrea
  • Posts: 20
  • Joined: Aug 26, 2012
|
#6568
Hello… Hopefully someone can help me understand the setup to be able to see things quicker and arrive to the correct answers and the majority of the questions for the last game on the June 2012 LSAT game questions 19-23; The more I try to get the correct answers I just don’t understand how they arrive to those answers…. (Hopefully my questions and explanations do not confuse anyone)

Option 1:
W w/z w/z Z
M T T M __
1 2 3 4 5

Option 2:
W z/w z/w W
M _ T T __
1 2 3 4 5



Question 20- Why would the minimum be zero when there is the possibility of having
W
T in placement 4 based the stated rules?


***I hope I have made sense, please get back to when possible, and most importantly out of this game, is I would really like to know how you would set it up on the first time to be able to see all inferences and move through the game quickly.

THANKS!!!
User avatar
 Dave Killoran
PowerScore Staff
  • PowerScore Staff
  • Posts: 5853
  • Joined: Mar 25, 2011
|
#6611
Hi Karla,

Again, this is about the question stem. The stem isn't trying to ascertain if it is possible for W to play a traditional piece, but how many traditional pieces he has to play. So, while W can play a traditional piece as stated in the third rule, it is possible that W never plays a traditional piece, and hence the answer is zero.

I will note that in both 19 and 20 you had the understanding of what was occurring (at least I think so); it was the way the question was worded that tripped you up. That's an easily correctable thing, so that's a good sign :-D

Please let me know if that helps. Thanks!

Get the most out of your LSAT Prep Plus subscription.

Analyze and track your performance with our Testing and Analytics Package.